What values determine the concavity of a parametric curve?

  • Thread starter Thread starter sapiental
  • Start date Start date
  • Tags Tags
    Curve Parametric
Click For Summary
To determine the concavity of a parametric curve, the second derivative is calculated, as shown with the example x = t^2 and y = t^3 - 3t. The second derivative, d²y/dx² = (3(t² + 1))/(4t³), indicates that the curve is concave upward when t > 0 and concave downward when t < 0. This conclusion arises because the numerator is always positive, while the denominator is positive only for t > 0. Thus, the general procedure involves solving inequalities related to the second derivative to assess concavity. Understanding these values is essential for analyzing the behavior of parametric curves.
sapiental
Messages
110
Reaction score
0
Hello,

My textbook says that to determine concavity we calculate the second derivative of the curve. This is a problem from my book,

x = t^2 and y = t^3 - 3t

the second derivative of this is (3(t^2+1))/(4t^3)

I know all the steps to get to this point.. However, the book says that the curve is concave upward when t > 0 and concave downward when t < 0.

Can somebody please explain to me what values this last statement refers to. Is there a general theorem/procedure that I can apply to any second derivative of a parametric curve to determine the concavity?

Thanks a lot in advance.
 
Physics news on Phys.org
If f&#039;&#039;(x) &gt; 0 then the curve is concave upward and vice versa.Looking at \frac{d^{2}y}{dx^{2}} = \frac{3(t^{2}+1)}{4t^{3}}

it is greater than 0 when both numerator and denominator are positive (cant do negative over negative because the numerator will always be positive).

So solve the following inequalities: 3t^{2} + 3 &gt; 0 and 4t^{3} &gt; 0. Clearly, all values of t work for the first inequality, but only positive values of t work for the second inequality.
 
Question: A clock's minute hand has length 4 and its hour hand has length 3. What is the distance between the tips at the moment when it is increasing most rapidly?(Putnam Exam Question) Answer: Making assumption that both the hands moves at constant angular velocities, the answer is ## \sqrt{7} .## But don't you think this assumption is somewhat doubtful and wrong?

Similar threads

Replies
2
Views
2K
  • · Replies 4 ·
Replies
4
Views
3K
Replies
2
Views
1K
  • · Replies 12 ·
Replies
12
Views
2K
  • · Replies 7 ·
Replies
7
Views
1K
  • · Replies 2 ·
Replies
2
Views
1K
Replies
11
Views
2K
  • · Replies 2 ·
Replies
2
Views
1K
  • · Replies 4 ·
Replies
4
Views
3K
  • · Replies 2 ·
Replies
2
Views
3K